Inscription / Connexion Nouveau Sujet
Niveau Loisir
Partager :

Fermat-Wiles n=3 - Tait - 1870

Posté par
fabo34
01-06-23 à 18:26

Bonjour à tous,

   retour sur le cas Fermat-Wiles x³+y³=z³ . Sur le wikipedia anglais, il est indiqué une démonstration d'un mathématicien anglais dénommé Tait, dans les années 1870, avec le lien vers sa publication :

Ca dit que si (x;y;z) est un triplet solution, alors (y(x^3+z^3) ; z(x^3-y^3) ; x(y^3+z^3)) est aussi un triplet solution. Effectivement, ça fonctionne, et c'est génial!

Il finit par "This furnishes an easy proof of the impossibility of finding two integers the sum of whose  cubes  is a cube"

Easy?   Une idée de l'argument final?

Posté par
fabo34
re : Fermat-Wiles n=3 - Tait - 1870 01-06-23 à 18:30

Le lien n'a pas fonctionné:

Posté par
Ulmiere
re : Fermat-Wiles n=3 - Tait - 1870 02-06-23 à 13:18

Pose X = x^3, Y = y^3 et Z = X + Y = x^3+z^3 et le nouveau triplet généré donne

(X-Y)(X+Y)(X-Y-1)(X-Y+1) = 0

Posté par
fabo34
re : Fermat-Wiles n=3 - Tait - 1870 02-06-23 à 15:18

Bonjour Ulmiere.

Désolé, là je n'y suis pas. Pourrais-tu expliquer avec un peu plus de détails, s'il te plait?

Posté par
Sylvieg Moderateur
re : Fermat-Wiles n=3 - Tait - 1870 02-06-23 à 20:44

Bonsoir,
je pense qu'il y a une coquille dans le message d'Ulmiere.
Si X = x^3, Y = y^3 et Z = X + Y
alors Z = z^3.

Posté par
fabo34
re : Fermat-Wiles n=3 - Tait - 1870 02-06-23 à 21:09

Bonsoir!
Ah. Dans ce cas, un logiciel me donne cette remarquable factorisation du "nouveau" triplet:

Y(X+Z)^3+Z(X-Y)^3-X(Y+Z)^3=(X-Y)(X+Z)(Y+Z)(X+Y-Z)

Qui est bien nul avec  X+Y-Z=0

Incroyable, cette factorisation!
Cela dit, comment utiliser ça pour Fermat?

Posté par
fabo34
re : Fermat-Wiles n=3 - Tait - 1870 10-06-23 à 14:16

Bonjour à tous.

Je me permets de vous relancer sur ce sujet. Preuve "facile" ou boutade de Tait du genre "marge trop petite!"

Ulmiere, quelle était ton idée?

Si on travaille dans \mathbb{N}, alors z>x, z>y
On sait par modulo 8 qu'il y a forcément 1 pair dans (x,y) et z impair.
Ici c'est donc forcément y ~pair, qu'on notera y_p

Ainsi (y_p,x,z) \rightarrow  (y_p(x^3+z^3)>y_p, z(x^3-y^3)>x, x(y_p^3+z^3)>z)

Les termes du nouveau triplet  sont donc tous supérieurs à leur précécesseurs! Par contre, on voit apparaître la contrainte z>x>y_p afin de  rester dans \mathbb{N}. Une contradiction vers +\infty ?

Voilà, c'est très maigre de mon côté. Rien d'"easy" mais j'imagine que c'est normal, ça doit faire appel à des outils du Supérieur.

Aux spécialistes d'arithmétique, que penser de ce problème?
Y-a-t-il une solution?
Est-ce que ça vient contredire un résulat connu en 1870, genre "il n'existe pas une infinité de solution pour n=3" ?



Vous devez être membre accéder à ce service...

Pas encore inscrit ?

1 compte par personne, multi-compte interdit !

Ou identifiez-vous :


Rester sur la page

Inscription gratuite

Fiches en rapport

parmi 1674 fiches de maths

Désolé, votre version d'Internet Explorer est plus que périmée ! Merci de le mettre à jour ou de télécharger Firefox ou Google Chrome pour utiliser le site. Votre ordinateur vous remerciera !